AHZolfaghari

Well-Known Member
ارسال ها
935
لایک ها
1,654
امتیاز
93
پاسخ : ماراتن جبر (سطح پیشرفته)

ببین ا.ذ. هم گفت تابع
هم صدق میکنه خب ایده ی قبلیمو میتونیم تعمیم بدیم و بگیم تابع
هم صدق میکنه اما میدونیم فقط به ازای
صحیحه خب پس حل من ناقصه و خودمم میدونم فقط میخوام اشکالشو پیدا کنم !!! ببخشید این سوال اخر حل نشد بگید مال کجاست البته راحت ازش نگذریم سوال خوبیه اقای ا.ذ. خودتون اگه راه حل دارید بذار !!!
سوال جایی نیست !!! سوالی هست که طرح یک دانش آموز هستش !! حلشو بلد نیستم خودم .تو مث لینکس هم گذاشتم تقریبا 400-500 view داشته اما کسی جوابشو نذاشته . هرچند چندبار اونجا گفتم که بیاید جواب بدید و من نیاز به جواب این سوال دارم و ... اما کسی بلد نبوده !!
 

darya.f

New Member
ارسال ها
182
لایک ها
114
امتیاز
0
پاسخ : ماراتن جبر (سطح پیشرفته)

سوال جایی نیست !!! سوالی هست که طرح یک دانش آموز هستش !! حلشو بلد نیستم خودم .تو مث لینکس هم گذاشتم تقریبا 400-500 view داشته اما کسی جوابشو نذاشته . هرچند چندبار اونجا گفتم که بیاید جواب بدید و من نیاز به جواب این سوال دارم و ... اما کسی بلد نبوده !!
ىکم فرصت بدىن...فک کردم..ىه چزاى خوبى درومده..اىشالا که حل شه؛)
 

Dadgarnia

New Member
ارسال ها
1,350
لایک ها
1,127
امتیاز
0
پاسخ : ماراتن جبر (سطح پیشرفته)

همونطور که آقای ذوالفقاری فرموده بودند سوالشون تاپیک رو خوابوند. حالا من سوال بعد رو میذارم هر کی سوال ایشون رو حل کرد جوابش رو همین جا بنویسه:
تمام چند جمله ای های
را بیابید به طوریکه برای هر
داشته باشیم:
 

AHZolfaghari

Well-Known Member
ارسال ها
935
لایک ها
1,654
امتیاز
93
پاسخ : ماراتن جبر (سطح پیشرفته)

اون سوالی که من گذاشتم صورت درست طرف چپش اینه :
f(x^3) + f(y^3) = s
در واقع منظورم این بود .
هرچند ظاهرا این سوالی هم که من به اشتباه گذاشتم هم میتونه سوال خوبی برای تفکر باشه !!

---- دو نوشته به هم متصل شده است ----

اون سوالی که من گذاشتم صورت درست طرف چپش اینه :
f(x^3) + f(y^3) = s
در واقع منظورم این بود .
هرچند ظاهرا این سوالی هم که من به اشتباه گذاشتم هم میتونه سوال خوبی برای تفکر باشه !!
 

aras2213

New Member
ارسال ها
216
لایک ها
228
امتیاز
0
پاسخ : ماراتن جبر (سطح پیشرفته)

همونطور که آقای ذوالفقاری فرموده بودند سوالشون تاپیک رو خوابوند. حالا من سوال بعد رو میذارم هر کی سوال ایشون رو حل کرد جوابش رو همین جا بنویسه:
تمام چند جمله ای های
را بیابید به طوریکه برای هر
داشته باشیم:
اول اگه 2010 رو تو معادله قرار بدیم نتیجه میشه که
،از طرفی اگه 67-2010 رو قرار بدیم نتییجه میشه
.همجین جور ادامه میدیم چون
پس
.با جایگذاری
.پس
.

---- دو نوشته به هم متصل شده است ----

سوال بعد:
و
دو چندجمله ای تکین با ضرایب حقیقی هستند و
.نشان دهید اگر معادله ی
هیچ جوابی در اعداد حقیقی نداشته باشد آن گاه معادله
حداقل یک جواب حقیقی دارد.
 

Dadgarnia

New Member
ارسال ها
1,350
لایک ها
1,127
امتیاز
0
پاسخ : ماراتن جبر (سطح پیشرفته)

اول اگه 2010 رو تو معادله قرار بدیم نتیجه میشه که
،از طرفی اگه 67-2010 رو قرار بدیم نتییجه میشه
.همجین جور ادامه میدیم چون
پس
.با جایگذاری
.پس
.

---- دو نوشته به هم متصل شده است ----

سوال بعد:
و
دو چندجمله ای تکین با ضرایب حقیقی هستند و
.نشان دهید اگر معادله ی
هیچ جوابی در اعداد حقیقی نداشته باشد آن گاه معادله
حداقل یک جواب حقیقی دارد.
فرض مي كنيم
با توجه به فرض سوال مي دونيم كه
هيچ ريشه ي حقيقي نداره پس درجه اش بايد زوج باشه و داريم
. حالا اگه تعريف كنيم
به راحتي مي توانيم بدست بياريم
پس
يه چند جمله اي از درجه ي فرده و با توجه به قضيه ي مقدار مياني حداقل يه ريشه ي حقيقي داره و چيزي كه مي خواستيم ثابت شد.

---- دو نوشته به هم متصل شده است ----

سوال بعد:
تمام چند جمله اي هاي با ضرايب حقيقي را بيابيد كه داشته باشيم
 

math1998

New Member
ارسال ها
336
لایک ها
224
امتیاز
0
پاسخ : ماراتن جبر (سطح پیشرفته)

فرض مي كنيم
با توجه به فرض سوال مي دونيم كه
هيچ ريشه ي حقيقي نداره پس درجه اش بايد زوج باشه و داريم
. حالا اگه تعريف كنيم
به راحتي مي توانيم بدست بياريم
پس
يه چند جمله اي از درجه ي فرده و با توجه به قضيه ي مقدار مياني حداقل يه ريشه ي حقيقي داره و چيزي كه مي خواستيم ثابت شد.

---- دو نوشته به هم متصل شده است ----

سوال بعد:
تمام چند جمله اي هاي با ضرايب حقيقي را بيابيد كه داشته باشيم









سوال بعد : اگر
ثابت کنید :



 
آخرین ویرایش توسط مدیر

Dadgarnia

New Member
ارسال ها
1,350
لایک ها
1,127
امتیاز
0
پاسخ : ماراتن جبر (سطح پیشرفته)

سوال بعد:
تمام توابع
را بيابيد به طوريكه براي هر
داشته باشيم:
 

math1998

New Member
ارسال ها
336
لایک ها
224
امتیاز
0
پاسخ : ماراتن جبر (سطح پیشرفته)

سوال بعد:
تمام توابع
را بيابيد به طوريكه براي هر
داشته باشيم:






پس از ساده سازی و تشکیل معادله درجه 2



 

Dadgarnia

New Member
ارسال ها
1,350
لایک ها
1,127
امتیاز
0
پاسخ : ماراتن جبر (سطح پیشرفته)







پس از ساده سازی و تشکیل معادله درجه 2



توي خط سوم يه اشتباهي كردين درستش اينه
و جواب آخرتون هم اشتباهه.
 
آخرین ویرایش توسط مدیر

math1998

New Member
ارسال ها
336
لایک ها
224
امتیاز
0
پاسخ : ماراتن جبر (سطح پیشرفته)

توي خط سوم يه اشتباهي كردين درستش اينه
و جواب آخرتون هم اشتباهه.
بله اینجا حواسم نبود اما در کل من دوبار جوابو حساب کردم
بر حسب
بدست میاد!!!!
 

Dadgarnia

New Member
ارسال ها
1,350
لایک ها
1,127
امتیاز
0
پاسخ : ماراتن جبر (سطح پیشرفته)

بله اینجا حواسم نبود اما در کل من دوبار جوابو حساب کردم
بر حسب
بدست میاد!!!!
بله درسته و بقيه اش ديگه آسونه ولي جوابش ميشه
و براي g هم دو تا جواب بدست مياد
 

math1998

New Member
ارسال ها
336
لایک ها
224
امتیاز
0
پاسخ : ماراتن جبر (سطح پیشرفته)

بله درسته و بقيه اش ديگه آسونه ولي جوابش ميشه
و براي g هم دو تا جواب بدست مياد
اره خوب درسته منم منظورم همین بود فقط دیگه معادله درجه 2 رو حساب نکردم اما در کل منظورم تابع ثابت بود!!!!
 

Dadgarnia

New Member
ارسال ها
1,350
لایک ها
1,127
امتیاز
0
پاسخ : ماراتن جبر (سطح پیشرفته)

سوال بعد:
ثابت كنيد
تجزيه ناپذيره براي هر n بزرگتر يا مساوي صفر.
 

aras2213

New Member
ارسال ها
216
لایک ها
228
امتیاز
0
پاسخ : ماراتن جبر (سطح پیشرفته)

فرض کنید که
.حالا این معادله رو به پیمانه 2 نگاه کنید.چون که
پس بدست میاد که
.حالا چون عبارت داخل پرانتز تحویل ناپذیره،
.

در نتیجه
.

پس
.حالا اگه
،
.اما P,Q

دارای ضرایبی صحیح هستند پس
به شکل
هست ولی 1/2 این جوری نیست.
 

Dadgarnia

New Member
ارسال ها
1,350
لایک ها
1,127
امتیاز
0
پاسخ : ماراتن جبر (سطح پیشرفته)

فرض کنید که
.حالا این معادله رو به پیمانه 2 نگاه کنید.چون که
پس بدست میاد که
.حالا چون عبارت داخل پرانتز تحویل ناپذیره،
.

در نتیجه
.

پس
.حالا اگه
،
.اما P,Q

دارای ضرایبی صحیح هستند پس
به شکل
هست ولی 1/2 این جوری نیست.
يعني چي و چه خواصي داره؟
 

aras2213

New Member
ارسال ها
216
لایک ها
228
امتیاز
0
پاسخ : ماراتن جبر (سطح پیشرفته)

يعني چي و چه خواصي داره؟
سوال خوبیه:4:(من هم اولین بار که دیدم اینا رو نفهمیدم، بعد از استاد شاولی پرسیدم توضیح دادند برام)
همون
که ضرایبش به پیمانه 2 حساب میشن.مثلا اگر
، اون وقت
برابر با
میشه.حالا خاصیت مهم این کاری که ما انجام میدیم اینه که اگه معادله
برقرار باشه اون وقت اگر دو طرف معادله رو به پیمانه p اول نگاه کنیم باز هم معادله برقراره.
برای مثال لم آیزنشتاین رو میتونید این جوری ثابت کنید.

---- دو نوشته به هم متصل شده است ----

سوال بعد:نشان دهید چندجمله ایه
که p عددی اول است و
تحویل ناپذیر است.
 

Dadgarnia

New Member
ارسال ها
1,350
لایک ها
1,127
امتیاز
0
پاسخ : ماراتن جبر (سطح پیشرفته)

سوال بعد:نشان دهید چندجمله ایه
که p عددی اول است و
تحویل ناپذیر است.
نمي دونم راهم درسته يا نه همچنين از يكي از فرض هاي سوال هم استفاده نكردم ولي مي نويسم تا تاپيك بالا بياد.
فرض مي كنيم چند جمله اي هاي صحيح الضرايب
موجود باشند كه
پس داريم (اين نمادي كه در ادامه مشاهده خواهيد كرد ابداعي خودمه :4: فكر مي كنم معنا و مفهومش با توجه به پست آقا ارس مشخص باشه ولي اگه نفهميدن مي تونم همينجا بنويسم):
پس اعداد صحيح نامنفي
و چند جمله اي هاي صحيح الضرايب
وجود دارند كه:
مي دونيم كه
پس
. اگر
باشند با جايگذاري اين روابط در صورت سوال داريم:
حالا بنابر تقارن فرض مي كنيم
صفر باشه و
در اين حالت داريم:
پس
هست. با جايگذاري توي صورت سوال داريم:
حالا با بررسي شرايط لم آيزنشتاين مي تونيم ببينيم كه اين چند جمله اي تحويل نا پذيره.

---- دو نوشته به هم متصل شده است ----

اگه درسته اينم سوال بعد:
تمام توابع
را بيابيد به طوريكه براي هر
داشته باشيم:

 

aras2213

New Member
ارسال ها
216
لایک ها
228
امتیاز
0
پاسخ : ماراتن جبر (سطح پیشرفته)

نمي دونم راهم درسته يا نه همچنين از يكي از فرض هاي سوال هم استفاده نكردم ولي مي نويسم تا تاپيك بالا بياد.
فرض مي كنيم چند جمله اي هاي صحيح الضرايب
موجود باشند كه
پس داريم (اين نمادي كه در ادامه مشاهده خواهيد كرد ابداعي خودمه :4: فكر مي كنم معنا و مفهومش با توجه به پست آقا ارس مشخص باشه ولي اگه نفهميدن مي تونم همينجا بنويسم):
پس اعداد صحيح نامنفي
و چند جمله اي هاي صحيح الضرايب
وجود دارند كه:
مي دونيم كه
پس
. اگر
باشند با جايگذاري اين روابط در صورت سوال داريم:
حالا بنابر تقارن فرض مي كنيم
صفر باشه و
در اين حالت داريم:
پس
هست. با جايگذاري توي صورت سوال داريم:
حالا با بررسي شرايط لم آيزنشتاين مي تونيم ببينيم كه اين چند جمله اي تحويل نا پذيره.

---- دو نوشته به هم متصل شده است ----

اگه درسته اينم سوال بعد:
تمام توابع
را بيابيد به طوريكه براي هر
داشته باشيم:

ابتدا f پوشا و یک به یک است.پس u وجود دارد که f(u)=0 .در این صورت:


حالا
.از طرفی
که از مقایسه

این دو داریم:
.همچنین از مقایسه این دو
و با توجه به 1-1 بودن نتیجه میشود که



حالا داریم
.از مقایسه این با قبلی نتیجه میشه که
.
.حالا برای

عدد حقیقی و دلخواه x_0 با توجه به پوشا بودن f ، عددی مثل y وجود دارد که
.در این صورت
. از طرفی
.با مقایسه این با قبلی نتیجه میشه که

.
اگر هم
باشد در این صورت نتیجه میشود
که جواب نیست.پس تنها جواب
است.

آیا جوب داره؟:223:
 
آخرین ویرایش توسط مدیر
بالا